A nonidentity permutation $sigma$ satisfies $sigma(i)<i$ for some $i$ [closed]












0












$begingroup$


If a permutation $sigma: Nto N$ is not the identity, prove that there exists an $i in {1,...,n} $ such that $sigma(i)<i$.










share|cite|improve this question











$endgroup$



closed as off-topic by caverac, user10354138, Lord Shark the Unknown, Rebellos, Vidyanshu Mishra Dec 5 '18 at 7:27


This question appears to be off-topic. The users who voted to close gave this specific reason:


  • "This question is missing context or other details: Please provide additional context, which ideally explains why the question is relevant to you and our community. Some forms of context include: background and motivation, relevant definitions, source, possible strategies, your current progress, why the question is interesting or important, etc." – caverac, user10354138, Rebellos, Vidyanshu Mishra

If this question can be reworded to fit the rules in the help center, please edit the question.





















    0












    $begingroup$


    If a permutation $sigma: Nto N$ is not the identity, prove that there exists an $i in {1,...,n} $ such that $sigma(i)<i$.










    share|cite|improve this question











    $endgroup$



    closed as off-topic by caverac, user10354138, Lord Shark the Unknown, Rebellos, Vidyanshu Mishra Dec 5 '18 at 7:27


    This question appears to be off-topic. The users who voted to close gave this specific reason:


    • "This question is missing context or other details: Please provide additional context, which ideally explains why the question is relevant to you and our community. Some forms of context include: background and motivation, relevant definitions, source, possible strategies, your current progress, why the question is interesting or important, etc." – caverac, user10354138, Rebellos, Vidyanshu Mishra

    If this question can be reworded to fit the rules in the help center, please edit the question.



















      0












      0








      0





      $begingroup$


      If a permutation $sigma: Nto N$ is not the identity, prove that there exists an $i in {1,...,n} $ such that $sigma(i)<i$.










      share|cite|improve this question











      $endgroup$




      If a permutation $sigma: Nto N$ is not the identity, prove that there exists an $i in {1,...,n} $ such that $sigma(i)<i$.







      abstract-algebra functions permutations






      share|cite|improve this question















      share|cite|improve this question













      share|cite|improve this question




      share|cite|improve this question








      edited Dec 9 '18 at 16:11









      Matt Samuel

      38k63666




      38k63666










      asked Dec 5 '18 at 0:57









      cercer

      405




      405




      closed as off-topic by caverac, user10354138, Lord Shark the Unknown, Rebellos, Vidyanshu Mishra Dec 5 '18 at 7:27


      This question appears to be off-topic. The users who voted to close gave this specific reason:


      • "This question is missing context or other details: Please provide additional context, which ideally explains why the question is relevant to you and our community. Some forms of context include: background and motivation, relevant definitions, source, possible strategies, your current progress, why the question is interesting or important, etc." – caverac, user10354138, Rebellos, Vidyanshu Mishra

      If this question can be reworded to fit the rules in the help center, please edit the question.







      closed as off-topic by caverac, user10354138, Lord Shark the Unknown, Rebellos, Vidyanshu Mishra Dec 5 '18 at 7:27


      This question appears to be off-topic. The users who voted to close gave this specific reason:


      • "This question is missing context or other details: Please provide additional context, which ideally explains why the question is relevant to you and our community. Some forms of context include: background and motivation, relevant definitions, source, possible strategies, your current progress, why the question is interesting or important, etc." – caverac, user10354138, Rebellos, Vidyanshu Mishra

      If this question can be reworded to fit the rules in the help center, please edit the question.






















          2 Answers
          2






          active

          oldest

          votes


















          2












          $begingroup$

          We prove that if $sigma(i)geq i$ for all $i$, then $sigma$ is the identity. We do this by induction on $n$. Clearly it is true if $n=1$. Otherwise, note that $sigma(n)geq n$, hence $sigma(n)=n$ since $n$ is the largest element. Then $sigma$ restricted to $[n-1]$ is a permutation with the same property on the smaller set, hence is the identity.






          share|cite|improve this answer









          $endgroup$





















            1












            $begingroup$

            Suppose the contrary, that $sigma (i)ge i,,forall i$. Then there is a minimum $hat i$ for which $sigma (hat i)gthat i$. Now the only remaining choices for $sigma ^{-1}(hat i)$ are all greater than $hat i$. So if $sigma (k)=hat i$, then $kgtsigma (k)$.






            share|cite|improve this answer









            $endgroup$




















              2 Answers
              2






              active

              oldest

              votes








              2 Answers
              2






              active

              oldest

              votes









              active

              oldest

              votes






              active

              oldest

              votes









              2












              $begingroup$

              We prove that if $sigma(i)geq i$ for all $i$, then $sigma$ is the identity. We do this by induction on $n$. Clearly it is true if $n=1$. Otherwise, note that $sigma(n)geq n$, hence $sigma(n)=n$ since $n$ is the largest element. Then $sigma$ restricted to $[n-1]$ is a permutation with the same property on the smaller set, hence is the identity.






              share|cite|improve this answer









              $endgroup$


















                2












                $begingroup$

                We prove that if $sigma(i)geq i$ for all $i$, then $sigma$ is the identity. We do this by induction on $n$. Clearly it is true if $n=1$. Otherwise, note that $sigma(n)geq n$, hence $sigma(n)=n$ since $n$ is the largest element. Then $sigma$ restricted to $[n-1]$ is a permutation with the same property on the smaller set, hence is the identity.






                share|cite|improve this answer









                $endgroup$
















                  2












                  2








                  2





                  $begingroup$

                  We prove that if $sigma(i)geq i$ for all $i$, then $sigma$ is the identity. We do this by induction on $n$. Clearly it is true if $n=1$. Otherwise, note that $sigma(n)geq n$, hence $sigma(n)=n$ since $n$ is the largest element. Then $sigma$ restricted to $[n-1]$ is a permutation with the same property on the smaller set, hence is the identity.






                  share|cite|improve this answer









                  $endgroup$



                  We prove that if $sigma(i)geq i$ for all $i$, then $sigma$ is the identity. We do this by induction on $n$. Clearly it is true if $n=1$. Otherwise, note that $sigma(n)geq n$, hence $sigma(n)=n$ since $n$ is the largest element. Then $sigma$ restricted to $[n-1]$ is a permutation with the same property on the smaller set, hence is the identity.







                  share|cite|improve this answer












                  share|cite|improve this answer



                  share|cite|improve this answer










                  answered Dec 5 '18 at 1:55









                  Matt SamuelMatt Samuel

                  38k63666




                  38k63666























                      1












                      $begingroup$

                      Suppose the contrary, that $sigma (i)ge i,,forall i$. Then there is a minimum $hat i$ for which $sigma (hat i)gthat i$. Now the only remaining choices for $sigma ^{-1}(hat i)$ are all greater than $hat i$. So if $sigma (k)=hat i$, then $kgtsigma (k)$.






                      share|cite|improve this answer









                      $endgroup$


















                        1












                        $begingroup$

                        Suppose the contrary, that $sigma (i)ge i,,forall i$. Then there is a minimum $hat i$ for which $sigma (hat i)gthat i$. Now the only remaining choices for $sigma ^{-1}(hat i)$ are all greater than $hat i$. So if $sigma (k)=hat i$, then $kgtsigma (k)$.






                        share|cite|improve this answer









                        $endgroup$
















                          1












                          1








                          1





                          $begingroup$

                          Suppose the contrary, that $sigma (i)ge i,,forall i$. Then there is a minimum $hat i$ for which $sigma (hat i)gthat i$. Now the only remaining choices for $sigma ^{-1}(hat i)$ are all greater than $hat i$. So if $sigma (k)=hat i$, then $kgtsigma (k)$.






                          share|cite|improve this answer









                          $endgroup$



                          Suppose the contrary, that $sigma (i)ge i,,forall i$. Then there is a minimum $hat i$ for which $sigma (hat i)gthat i$. Now the only remaining choices for $sigma ^{-1}(hat i)$ are all greater than $hat i$. So if $sigma (k)=hat i$, then $kgtsigma (k)$.







                          share|cite|improve this answer












                          share|cite|improve this answer



                          share|cite|improve this answer










                          answered Dec 5 '18 at 2:02









                          Chris CusterChris Custer

                          12.5k3825




                          12.5k3825















                              Popular posts from this blog

                              Plaza Victoria

                              In PowerPoint, is there a keyboard shortcut for bulleted / numbered list?

                              How to put 3 figures in Latex with 2 figures side by side and 1 below these side by side images but in...